- Wed Oct 03, 2012 11:00 pm
#41265
Complete Question Explanation
(The complete setup for this game can be found here: lsat/viewtopic.php?t=6641)
The correct answer choice is (C)
Answer choice (A) is incorrect because the first piece cannot be a T in order to conform to the fifth rule.
Answer choice (B) is incorrect because it violates the second rule by having three Ts in a row.
Answer choice (C) is the correct answer, and is possible as shown in Template #1.
Answer choice (D) is incorrect because it violates the second rule by not having two Ts in a row.
Answer choice (E) is incorrect because it violates the first rule.
(The complete setup for this game can be found here: lsat/viewtopic.php?t=6641)
The correct answer choice is (C)
Answer choice (A) is incorrect because the first piece cannot be a T in order to conform to the fifth rule.
Answer choice (B) is incorrect because it violates the second rule by having three Ts in a row.
Answer choice (C) is the correct answer, and is possible as shown in Template #1.
Answer choice (D) is incorrect because it violates the second rule by not having two Ts in a row.
Answer choice (E) is incorrect because it violates the first rule.